Math Pro 數學補給站's Archiver

少林寺的和尚武功千變萬化、飛簷走壁,
是過去挑了多少桶水上山?

bugmens 發表於 2011-5-15 21:02

100中科實中

以下資料供以後的考生參考:

初試最低錄取分數 52分
1名正式教師,取8名參加複試
68,65,63,53,52,52,52,52

其他,
51分     2人
40~49分 17人
30~39分 39人
20~29分 46人
10~19分 51人
  0~ 9分 26人
缺考     6人

共計 195 人
(准考證號碼從1001030151~1001030180無資料)

h ttp://140.120.84.1/01news/new_01_main.php?bull_id=1948&cate_id=203 連結已失效
發布單位: 教務處
發布日期: 2011-05-16
公告主旨:重新公告數學科及音樂科初試成績。
公告說明:
一、依據本校100年5月16日受理成績復查結果辦理。
二、數學科:100103029成績複查後初試成績修正為52分。
三、音樂科:選擇題第15題經命題、閱卷委員等研商、考究相關文獻後,議定:選擇答案B或答案C,視為正確答案均給分,並全面啟動重新閱卷後,重新公告初試成績。
四、詳請參閱附加檔案。

bugmens 發表於 2011-5-15 21:03

4.
\( \displaystyle \frac{3^4+2^6}{7^4+2^6}\times \frac{11^4+2^6}{15^4+2^6} \times \frac{19^4+2^6}{23^4+2^6}\times \frac{27^4+2^6}{31^4+2^6}\times \frac{35^4+2^6}{39^4+2^6}\times \frac{43^4+2^6}{47^4+2^6} \)
[提示]
\( n^4+4 \times 2^4=[(n-2)^2+2^2][(n+2)^2+2^2] \)

Compute \( \displaystyle \frac{(10^4+324)(22^4+324)(34^4+324)(46^4+324)(58^4+324)}{(4^4+324)(16^4+324)(28^4+324)(40^4+324)(52^4+324)} \).
(1987AIME)
這裡還有類似的題目
[url]https://math.pro/db/viewthread.php?tid=1041&page=1#pid2840[/url]



6.
若從1,2,...,13中任選出相異三數\( x,y,z \),且\( x<y<z \),則\( y-x \ge 3 \)且\( z-y \ge 3 \)成立之機率為
[解答]
\( a=13-z \ge 0 \)
\( b=x-1 \ge 0 \)
\( c=y-x-3 \ge 0 \)
\( d=z-y-3 \ge 0 \)
四式相加得
\( a+b+c+d=6 \)
求非負整數解的個數有\( \displaystyle H_6^4 \)種

滿足\( 1 \le a \le b < c \le d \le 8 \)的整數解\( (a,b,c,d) \)共有幾組?
(95新竹高商)
\( x_1=a-1 \ge 0 \)
\( x_2=b-a \ge 0 \)
\( x_3=c-b-1 \ge 0 \)
\( x_4=d-c \ge 0 \)
\( x_5=8-d \ge 0 \)
五式相加得
\( x_1+x_2+x_3+x_4+x_5=6 \)
求非負整數解的個數有\( \displaystyle H_6^5 \)種



9.
\( \displaystyle \root 3 \of{\root 3 \of {2}-1}=\root 3 \of a+\root 3 \of b+\root 3 \of c \),其中\( a,b,c \in Q \)。求\( a+b+c= \)?
(92高中數學能力競賽,高中數學101 P25)
[url]https://math.pro/db/viewthread.php?tid=1041&page=1#pid2840[/url]



12.
\( sin \alpha+cos \beta=a \),\( cos \alpha+sin \beta=b \),求\( sin(\alpha-\beta) \)?(以\( a,b \)表示)

If \( sin x+cos y=a \) and \( cos x+sin y=b \). Find the value of \( \displaystyle tan (\; \frac{x-y}{2} )\; \) in terms of a and b.
[url]http://www.artofproblemsolving.com/Forum/viewtopic.php?f=150&t=356363[/url]

已知\( \displaystyle sin \alpha+cos \beta=\frac{3}{5} \),\( cos \alpha+sin \beta=\frac{4}{5} \)。求\( cos \alpha sin \beta \)的值?
(96家齊女中,[url]http://forum.nta.org.tw/examservice/showthread.php?t=23930[/url])
(99屏東女中,[url]https://math.pro/db/thread-976-1-1.html[/url])
[img]https://math.pro/db/attachment.php?aid=300&k=1346b166eb413c8c6bb4d8045067a489&t=1305544323&noupdate=yes[/img]



13.
4個A、4個B、4個C排成一列,第1到第4位置稱為Ⅰ區,第5到第8位置稱為Ⅱ區,第9到第12位置稱為Ⅲ區,若A不在Ⅰ區,B不在Ⅱ區,C不在Ⅲ區的排列方法有幾種?
[url]https://math.pro/db/thread-454-1-1.html[/url]

老王 發表於 2011-5-15 21:27

第六題
若從\(1,2,\ldots,13\)中任選出相異三數\(x\)、\(y\)、\(z\),且\(x<y<z\),則\(y-x\ge 3\)且\(z-y\ge 3\)成立之機率為[u]   [/u]。
[解答]
意思是x,y至少差2;y,z也至少差2
那麼x,y和y,z之間先各塞兩個數,還剩下13-3-4=6個數,可以放在x前面、x,y之間、y,z之間和z後面四個地方
所以是\(\displaystyle H_6^4 \)

第三題是怎樣??這種東西要記???我只知道上限是兩人,其他什麼單位??

第二題,我指導的學生,可是他這篇是物理的!!!!
我們學校,唉,好資值的學生都被先被其他科搶走,感嘆!!!!!!!!

老王 發表於 2011-5-15 21:44

計算證明一
設\(f(x)=cos x+sin(\sqrt{3}x)\),試證:\(f(x)\)不是週期函數。
[解答]
\(\displaystyle \cos{x} \)的週期是\(\displaystyle 2\pi \)
\(\displaystyle \sin{\sqrt{3}x} \)的週期是\(\displaystyle \frac{2\pi}{\sqrt3} \)
假設f(x)是週期函數,其週期為p
那麼存在正整數m,n,使得
\(\displaystyle p=m \times 2\pi , p=n \times \frac{2\pi}{\sqrt3} \)
兩式相除得到\(\displaystyle \sqrt3=\frac{n}{m} \)
那麼\(\displaystyle \sqrt3 \)為有理數,與已知矛盾,
故f(x)不是週期函數

老王 發表於 2011-5-15 23:25

計算證明二
對任意正整數\(n\),設\(a_n\)是方程\(\displaystyle x^3+\frac{x}{n}=1\)的正實數根,求證:(1)\(a_{n+1}>a_n\) (2)\(\displaystyle \sum_{i=1}^n \frac{1}{(i+1)^2a_i}<1\)。
[解答]
(1)
顯然所有\( a_n \)在(0,1)之間
\(\displaystyle a_{n+1}^3+\frac{a_{n+1}}{n+1}=1 \)
\(\displaystyle a_{n}^3+\frac{a_{n}}{n}=1 \)
兩式相減
\(\displaystyle (a_{n+1}^3-a_{n}^3)+\frac{a_{n+1}}{n+1}-\frac{a_{n}}{n}=0 \)
\(\displaystyle (a_{n+1}^3-a_{n}^3)+\frac{a_{n+1}}{n}-\frac{a_{n}}{n}=\frac{a_{n+1}}{n}-\frac{a_{n+1}}{n+1}>0 \)
\(\displaystyle (a_{n+1}-a_{n})(a_{n+1}^2+a_{n+1}a_{n}+a_{n}^2+\frac{1}{n})>0 \)
後項為正,故
\(\displaystyle a_{n+1}-a_{n}>0 \)

(2)
實在找不到如何用第一小題來證,還是說其實我的第一小題不該這樣證。
看起來像積分,但是找不到,只好硬作。

先證明
\(\displaystyle a_{k}>\frac{k}{k+1} \)
就以\( x=\frac{k}{k+1} \)代入 \( x^3+\frac{x}{k}-1 \)
\(\displaystyle \frac{k^3}{(k+1)^3}+\frac{1}{k+1}-1 \)
只算分子部分
\(\displaystyle k^3+(k+1)^2-(k+1)^3=-2k^2-k<0 \)
故成立

接著就有
\(\displaystyle \frac{1}{(i+1)^2 a_{i}}<\frac{1}{i(i+1)}=\frac{1}{i}-\frac{1}{i+1} \)
所求就為
\(\displaystyle 1-\frac{1}{n+1}<1 \)

Fermat 發表於 2011-5-16 00:21

[quote]原帖由 [i]老王[/i] 於 2011-5-15 11:25 PM 發表 [url=https://math.pro/db/redirect.php?goto=findpost&pid=3152&ptid=1107][img]https://math.pro/db/images/common/back.gif[/img][/url]
計算證明二
(1)
顯然所有\( a_n \)在(0,1)之間
\(\displaystyle a_{n+1}^3+\frac{a_{n+1}}{n+1}=1 \)
\(\displaystyle a_{n}^3+\frac{a_{n}}{n}=1 \)
兩式相減... [/quote]

(1)我會用圖解來證

考慮
\(y=x^3-1\)與\(\displaystyle y=-\frac{x}{n}\)的交點的\(x\)坐標\(a_n\)
由圖知只有唯一一個交點, 且\(0<a_n<1\)
當\(n\)增加, 直線斜率\(\displaystyle -\frac{1}{n}\)遞增, 與\(y=x^3-1\)的交點\(x\)坐標\(a_n\)就遞增

waitpub 發表於 2011-5-16 18:41

請問一下填充第5題跟第10題

weiye 發表於 2011-5-16 20:34

填充第 10 題
一矩\(ABCD\)的周長為8,\(E\)為\(\overline{CD}\)的中點,一圓\(C\)過\(A\)、\(B\)兩點與\(\overline{CD}\)相切於\(E\)(如下圖),求圓\(C\)半徑的最小值為[u]   [/u]。
[解答]

設圓半徑為 \(r\) ,令如圖中的角度為 \(\theta,\)

[img]http://img852.imageshack.us/img852/3682/83824107.png[/img]

則 \(\overline{AD}=r+r\sin\theta, \overline{CD}=2r\cos\theta\)

已知 \(r+r\sin\theta+2r\cos\theta=4\)

\(\Rightarrow r\sin\theta+2r\cos\theta=4-r\)

所以 \(\left|4-r\right|\leq \sqrt{r^2+\left(2r\right)^2}\)

解得 \(r\geq-1+\sqrt{5}\) 或 \(r\leq-1-\sqrt{5}\)

且因為 \(r\) 為半徑,所以 \(r>0.\)

故,\(r\geq-1+\sqrt{5}.\)

亦即 \(r\) 的最小值為 \(-1+\sqrt{5}.\)

weiye 發表於 2011-5-16 20:55

填充題第 5 題
空間中有三點\(A(-1,1,3)\)、\(B(3,1,5)\)、\(P(4,-1,-4)\),若球面\(S\)過\(A\)、\(B\)兩點且球心在平面\(E\):\(5x-2y+5z-14=0\)上,則滿足此條件的球面\(S\)有無限多個,其中半徑最小的球面方程式為[u]   [/u]。
[解答]

球心必過 \(\overline{AB}\) 的垂直平分面,

先寫出 \(\overline{AB}\) 的垂直平分面為 \(2\cdot(x-1)+0\cdot(y-1)+1\cdot(z-4)=0\)

            \(\Rightarrow 2x+z-6=0\)

且依題意,球心亦在平面 \(E\) 上,

所以,可以先解出兩者的相交直線方程式的參數式,即為球心所在直線的的參數式

解兩面交線的參數式後,可設球心為 \(\displaystyle O(t,8-\frac{5}{2}t,6-2t)\)

則 \(\displaystyle \overline{OB}^2 = \left(t-3\right)^2+\left(8-\frac{5t}{2}-1\right)^2+\left(6-2t-5\right)^2 = \frac{45}{4}\left(t-2\right)^2+14\)

所以當 \(t=2\) 時,半徑最小為 \(\sqrt{14}\),

且此時球心坐標為 \((2,3,2)\)

故,所求球面方程式為 \(\left(x-2\right)^2+\left(y-3\right)^2+\left(z-2\right)^2=14.\)

hua77825 發表於 2011-5-17 00:25

可否請問一下各位老師們第7  14  15

感謝;)

Fermat 發表於 2011-5-17 08:28

[quote]原帖由 [i]hua77825[/i] 於 2011-5-17 12:25 AM 發表 [url=https://math.pro/db/redirect.php?goto=findpost&pid=3157&ptid=1107][img]https://math.pro/db/images/common/back.gif[/img][/url]
可否請問一下各位老師們第7  14  15

感謝;) [/quote]

7.
一袋中有5個球,分別寫上1、2、3、4、5號,今由其中任取一球記下其號碼後放回袋中,如此繼續\(n\)次,若\(P_n\)表紀錄到\(n\)次時數字和為偶數的機率,則\(\displaystyle \sum_{n=1}^{\infty}(\frac{1}{2}-P_n)=\)[u]   [/u]。
[解答]
先看出遞迴關係
P(n+1)=(2/5)P(n)+(3/5)[1-p(n)]
=> P(n+1)=(-1/5)P(n)+3/5
=> P(n+1)-1/2=(-1/5)[P(n)-1/2]
即< P(n)-1/2>為首項P(1)-1/2=2/5-1/2=-1/10, 公比-1/5的等比數列
原式= -Σ(n=1 to ∞) [p(n)-1/2]
=-(-1/10)/[1-(-1/5)]
=1/12

14.
如下圖(圖形中各線段之比例僅供參考,實際之比例敘述如後),
設\(\displaystyle \frac{\overline{AD}}{\overline{DB}}=\frac{1}{3}\)且\(\displaystyle \frac{\overline{BE}}{\overline{EC}}=\frac{1}{2}\)且\(\displaystyle \frac{\overline{AF}}{2}=\frac{\overline{FG}}{1}=\frac{\overline{GC}}{2}\),若\(\vec{BH}=\alpha \vec{BA}+\beta \vec{BC}\),則實數對\((\alpha,\beta)=\)[u]   [/u]。
[解答]
我的解法稍麻煩了些
以下BA表示向量BA, 餘類推
BH=αBA+βBC
設FH=tDF, GH=sEG, s,t為實數
BH=BF+FH
=(3/5)BA+(2/5)BC+t(BF-BD)
=(3/5)BA+(2/5)BC+t[(3/5)BA+(2/5)BC-(3/4)BA]
=[(12-3t)/20]BA+[(2+2t)/5]BC...(1)

BH=BG+GH
=(2/5)BA+(3/5)BC+s(BG-BE)
=(2/5)BA+(3/5)BC+s[(2/5)BA+(3/5)BC-(1/3)BC)]
=[(2+2s)/5]BA+[(9+4s)/15]BC...(2)
由(1)(2)知α=(12-3t)/20=(2+2s)/5, β=(2+2t)/5=(9+4s)/15
解得s=1/4, t=2/3(t可不解)
=> (α,β)=(1/2, 2/3)

15.
將一矩形(邊長均為整數)的角剪去一個三角形後形成一個新的五邊形,今知此五邊形之邊長為13,19,20,25,31(不一定照順序成五邊形),試問此五邊形之面積為[u]   [/u]。
[解答]
就一個長31, 寬25的矩形
在一角截去兩股為12(長的方向), 5(寬的方向)的直角三角形
則得五邊形五邊為31,25,19,13,20
即得五邊形面積=31*25-(1/2)*12*5=745

112.8.23補充
A pentagon is formed by cutting a triangular corner from a rectangular piece of paper. The five sides of the pentagon have lengths 13, 19, 20, 25 and 31, although this is not necessarily their order around the pentagon. The area of the pentagon is
(A)459 (B)600 (C)680 (D)720 (E)745
(1995AHSME,[url]https://artofproblemsolving.com/wiki/index.php/1995_AHSME_Problems/Problem_22[/url])

weiye 發表於 2011-5-17 08:48

第 14 題
如下圖(圖形中各線段之比例僅供參考,實際之比例敘述如後),
設\(\displaystyle \frac{\overline{AD}}{\overline{DB}}=\frac{1}{3}\)且\(\displaystyle \frac{\overline{BE}}{\overline{EC}}=\frac{1}{2}\)且\(\displaystyle \frac{\overline{AF}}{2}=\frac{\overline{FG}}{1}=\frac{\overline{GC}}{2}\),若\(\vec{BH}=\alpha \vec{BA}+\beta \vec{BC}\),則實數對\((\alpha,\beta)=\)[u]   [/u]。
[解答]
也可以坐標化,令 \(B(0,0), C(1,0), A(0,1)\)

然後用分點公式找出 \(D,E,F,G\) 點坐標,

再求 \(DF\) 直線與 \(EG\) 直線方程式,

並且找出兩直線的交點 \(H(\beta,  \alpha).\)

waitpub 發表於 2011-5-17 12:48

請問一下A座標為何可以定成(01)?

[quote]原帖由 [i]weiye[/i] 於 2011-5-17 08:48 AM 發表 [url=https://math.pro/db/redirect.php?goto=findpost&pid=3159&ptid=1107][img]https://math.pro/db/images/common/back.gif[/img][/url]
第 14 題也可以坐標化, [color=Red]令 \(B(0,0), C(1,0),A(0,1)\)[/color]

然後用分點公式找出 \(D,E,F,G\) 點坐標,

再求 \(DF\) 直線與 \(EG\) 直線方程式,

並且找出兩直線的交點 \(H(\beta,  \alpha).\) ... [/quote]

weiye 發表於 2011-5-17 16:23

回復 13# waitpub 的帖子

因為 \(\vec{BA}\) 與 \(\vec{BC}\) 不平行,

所以此兩向量線性獨立,

故可以當成此平面的基底向量。 ^__^

至於上面的坐標化,就是建立斜坐標。

hua77825 發表於 2011-5-18 01:30

回復 11# Fermat 的帖子

感謝Fermat 老師!!
也感謝weiye老師第14題的座標化,快了很多!!!

oscar 發表於 2011-5-19 00:17

第 7 題:
一袋中有5個球,分別寫上1、2、3、4、5號,今由其中任取一球記下其號碼後放回袋中,如此繼續\(n\)次,若\(P_n\)表紀錄到\(n\)次時數字和為偶數的機率,則\(\displaystyle \sum_{n=1}^{\infty}(\frac{1}{2}-P_n)=\)[u]   [/u]。
[解答]
假設 \(f(x)=(x + x^2 + x^3 + x^4 +x^5)^n\),則 \(P_n = \displaystyle\frac{f(x) 偶次項係數和}{5^n} = \displaystyle\frac{f(1) + f(-1)}{2 \times 5^n} = \displaystyle\frac{1}{2} + \displaystyle\frac{1}{2}(\displaystyle\frac{-1}{5})^n\)。

freekayikid 發表於 2011-5-19 23:25

第14題
如下圖(圖形中各線段之比例僅供參考,實際之比例敘述如後),
設\(\displaystyle \frac{\overline{AD}}{\overline{DB}}=\frac{1}{3}\)且\(\displaystyle \frac{\overline{BE}}{\overline{EC}}=\frac{1}{2}\)且\(\displaystyle \frac{\overline{AF}}{2}=\frac{\overline{FG}}{1}=\frac{\overline{GC}}{2}\),若\(\vec{BH}=\alpha \vec{BA}+\beta \vec{BC}\),則實數對\((\alpha,\beta)=\)[u]   [/u]。
[解答]
另解
假設BH與AC交於M
by孟氏定理
(AF/FM)*(MH/HB)*(BD/DA)=1  --------(1)
(CG/GM)*(MH/HB)*(BE/EC)=1  --------(2)
(1)/(2)  => GM/FM = 1/6  => GM:FM = 1:6
BM = 3/7BA + 4/7BC  (向量)
又by (1)  MH:HB = 1:7  => MH:MB=1:6
BC=7/6BM=7/6(3/7BA + 4/7BC)=1/2BA + 2/3BC  (向量)

tunmu 發表於 2011-5-27 11:57

想請問各位老師們第11題 ~~~謝謝大家!~~~

weiye 發表於 2011-5-27 13:04

回復 18# tunmu 的帖子

第 11 題
\(P\)為球面\(S\):\((x-1)^2+(y-2)^2+z^2=4\)上的動點,\(A(3,4,0)\)、\(B(3,3,2)\)為球面外兩點,求\(\overline{PA}^2+\overline{PB}^2\)的最大值為[u]   [/u]。
[解答]

先求出 \(A, B\) 的中點 \(\displaystyle C(3, \frac{7}{2}, 1)\)



\(\displaystyle \overline{AC}=\frac{\sqrt{5}}{2}\)

\(\displaystyle \overline{PC}\) 的最大值為 \(\displaystyle \sqrt{\left(1-3\right)^2+\left(2-\frac{7}{2}\right)^2+\left(0-1\right)^2}+2=\frac{\sqrt{29}}{2}+2\)

所以,\(\displaystyle \overline{PA}^2 + \overline{PB}^2= 2\left(\overline{PC}^2+\overline{AC}^2\right)\)

         \(\displaystyle \geq 2\left(\left(\frac{\sqrt{29}}{2}+2\right)^2+\frac{5}{4}\right)\)

         \(= 25+4\sqrt{29}.\)

liengpi 發表於 2011-5-27 15:18

可以請問第8題嗎
我想要換成積分的形式來處理
可是好像被積分的上下標困住了

請前輩賜教
感謝

頁: [1] 2

論壇程式使用 Discuz! Archiver   © 2001-2022 Comsenz Inc.